Problem

Suppose $f(x, y)=\frac{x}{y}, P=(0,2)$ and $\mathbf{v}=1 \mathbf{i}+1 \mathbf{j}$.
A. Find the gradient of $f$.
$(\nabla f)(x, y)=$ i+ $\mathrm{j}$

Note: Your answers should be expressions of $x$ and $y$; e.g. " $3 x-4 y "$
B. Find the gradient of $f$ at the point $P$.
$(\nabla f)(P)=$ i+

Note: Your answers should be numbers
C. Find the directional derivative of $f$ at $P$ in the direction of $\mathbf{v}$. $\left(D_{\mathbf{u}} f\right)(P)=$

Note: Your answer should be a number
D. Find the maximum rate of change of $f$ at $P$.

Note: Your answer should be a number
E. Find the (unit) direction vector $w$ in which the maximum rate of change occurs at $P$. $\mathbf{w}=\square \mathbf{i}+\square \mathbf{j}$

Note: Your answers should be numbers

Answer

Expert–verified
Hide Steps
Answer

The direction in which the maximum rate of change occurs at \(P\) is given by the direction of the gradient at \(P\). Since the gradient at \(P\) is \(\frac{1}{2} \mathbf{i}\), the direction vector is \(\mathbf{w} = 1 \mathbf{i} + 0 \mathbf{j}\)

Steps

Step 1 :Find the partial derivatives of \(f(x, y)\) with respect to \(x\) and \(y\): \(\frac{\partial f}{\partial x} = \frac{1}{y}\) and \(\frac{\partial f}{\partial y} = -\frac{x}{y^2}\)

Step 2 :The gradient of \(f\) is given by: \((\nabla f)(x, y) = \frac{1}{y} \mathbf{i} - \frac{x}{y^2} \mathbf{j}\)

Step 3 :Substitute \(x=0\) and \(y=2\) into the gradient to find the gradient of \(f\) at the point \(P=(0,2)\): \((\nabla f)(P) = \frac{1}{2} \mathbf{i}\)

Step 4 :Find the unit vector in the direction of \(\mathbf{v}=1 \mathbf{i}+1 \mathbf{j}\): \(\mathbf{u} = \frac{1}{\sqrt{2}} \mathbf{i} + \frac{1}{\sqrt{2}} \mathbf{j}\)

Step 5 :The directional derivative of \(f\) at \(P\) in the direction of \(\mathbf{v}\) is given by the dot product of the gradient of \(f\) at \(P\) and the unit vector in the direction of \(\mathbf{v}\): \(\left(D_{\mathbf{u}} f\right)(P) = \frac{1}{2\sqrt{2}}\)

Step 6 :The maximum rate of change of \(f\) at \(P\) is given by the magnitude of the gradient at \(P\): \(||(\nabla f)(P)|| = \frac{1}{2}\)

Step 7 :The direction in which the maximum rate of change occurs at \(P\) is given by the direction of the gradient at \(P\). Since the gradient at \(P\) is \(\frac{1}{2} \mathbf{i}\), the direction vector is \(\mathbf{w} = 1 \mathbf{i} + 0 \mathbf{j}\)

link_gpt